Understanding the Proof of x mod pq = y mod pq for Distinct Primes p and q

  • Thread starter Thread starter SpiffyEh
  • Start date Start date
  • Tags Tags
    Proof
Click For Summary
The discussion focuses on proving that if x mod p = y mod p and x mod q = y mod q for distinct primes p and q, then x mod pq = y mod pq. The proof relies on the fact that if p divides (x-y) and q divides (x-y), then their least common multiple, LCM(p,q), also divides (x-y). Since p and q are distinct primes, LCM(p,q) equals pq, leading to the conclusion that x is congruent to y modulo pq. Participants clarify the notation where p|x-y indicates that p divides (x-y), and they confirm that understanding the LCM is essential but ultimately simplifies to pq in this context. The conversation emphasizes that the relationship holds true without needing to delve deeply into LCM concepts.
SpiffyEh
Messages
191
Reaction score
0

Homework Statement



Show that given x mod p = y mod p and x mod q = y mod q, the following is true:
x mod pq = y mod pq.
p and q are distinct primes.


The Attempt at a Solution


Here is the proof from someone that I am trying to understand:

In general, x≡y (mod p) and x≡y (mod q) ⇒ x≡y (mod LCM(p,q)).
Proof. x≡y (mod p) and x≡y (mod q) implies p|x-y and q|x-y
implies LCM(p,q)|x-y, which means x≡y (mod LCM(p,q)). (Q.E.D.)
So, if p and q are different primes, x≡y (mod p) and x≡y (mod q) yield
x≡y (mod pq).


I do not understand the proof. Primarily what does p|x-y mean? or any notation with |. Also, how does the LCM(p,q) come into this?
 
Physics news on Phys.org
a|b means a devides b, which means that there exists an integer number n so that b = a*n
 
Oh ok. That makes more sense. I still don't understand how the LCM comes in but it's starting to come together some more. Thanks!
 
You're welcome. It's weird actually they brought the LCM up, because if p and q are primes, then LCM(p,q)=p*q
Hope that helps
 
Is there another way to prove this without LCM?
 
don't worry about the LCM. If p and q are primes, then LCM(p,q) = p*q, so it's the same thing!
 
Question: A clock's minute hand has length 4 and its hour hand has length 3. What is the distance between the tips at the moment when it is increasing most rapidly?(Putnam Exam Question) Answer: Making assumption that both the hands moves at constant angular velocities, the answer is ## \sqrt{7} .## But don't you think this assumption is somewhat doubtful and wrong?

Similar threads

Replies
30
Views
3K
  • · Replies 1 ·
Replies
1
Views
2K
  • · Replies 11 ·
Replies
11
Views
2K
  • · Replies 16 ·
Replies
16
Views
2K
Replies
4
Views
1K
  • · Replies 16 ·
Replies
16
Views
3K
  • · Replies 24 ·
Replies
24
Views
4K
  • · Replies 11 ·
Replies
11
Views
3K
  • · Replies 2 ·
Replies
2
Views
2K
  • · Replies 1 ·
Replies
1
Views
1K